0

If an anti-proton collides with a proton, they are supposed to totally annihilate each other, correct? Then how is conservation of momentum preserved?

Could you launch both protons and antiprotons toward a common point, where they annihilate, and give their energy back, while you keep the gained the momentum from the launch of the particles? This would allow for a closed engine and would seem to violate the conservation of momentum. What would prevent this from working?

  • 1
    The annihilation process produces photons in place of the particles in order to conserve both energy and momentum. See: https://en.wikipedia.org/wiki/Annihilation – tmwilson26 Nov 20 '15 at 20:31
  • @tmwilson26 That would be an answer to my question and I do not know why this is not a valid question (since it has a -1) –  Nov 20 '15 at 20:34
  • @tmwilson26 I thought that photons were massless and cannot "have" momentum. If photons can conserve momentum, why can't light be used for propulsion? –  Nov 20 '15 at 20:36
  • Photons do have momentum, but it is not $p=mv$, but instead $p=\hbar k$, where $k$ is the wavenumber of the photon. In principle in can be used for propulsion. See: http://physics.stackexchange.com/questions/112866/if-im-floating-in-space-and-i-turn-on-a-flashlight-will-i-accelerate – tmwilson26 Nov 20 '15 at 20:38
  • @tmwilson26 I learned in my physics class that it does not have momentum, so that is why I was thinking about this. Thank you for that answer. –  Nov 20 '15 at 20:39
  • As a note, I didn't downvote your question, but many of these types of questions have been asked and answered on this site or can be found with a google/wikipedia search. The users here tend to downvote questions that can be answered by using links from this site or wikipedia. Just a tip for the future. – tmwilson26 Nov 20 '15 at 20:40
  • When I searched my question, I didn't get an results. I thought it would be a good question to ahve on here, even if it could be found elsewhere. –  Nov 20 '15 at 20:42
  • 4
  • What makes you think momentum isn't conserved? There are many ways you could phrase pseudo-challenges to any of the conservation or other laws. Just because you challenge something doesn't make it a good question without some research to suport your challenge. – Bill N Nov 21 '15 at 13:28
  • @BillN I did not challenge it. It was a question. –  Nov 21 '15 at 14:15
  • @JohnRennie I saw that but I didn't really understand it because it is about "4-momentum" and is speaking about the number of photons necessary to conserve that. –  Nov 21 '15 at 14:17

0 Answers0